Expand f(z)=1/(z(z-3)) in a Laurent series valid for the indicated annular domain. a. b. c. 0

Related Book For  answer-question

Complex Variables and Applications

ISBN: 978-0073051949

8th edition

Authors: James Brown, Ruel Churchill

Posted Date: